Approximation of continuous functions by differentiable ones

Click For Summary
The discussion focuses on approximating a continuous function f with a defined function g, which is the average of f over an interval. Participants express confusion regarding the differentiability of g and its dependence on both x and δ. The first part of the problem asks to show that g is continuously differentiable, while the second part requires demonstrating that if f is uniformly continuous, the difference between f and g can be made arbitrarily small. Clarifications are sought on the implications of the integral's value and the absence of a limit in the problem statement. The conversation emphasizes the need for a deeper understanding of the properties of continuous functions and their approximations.
cooljosh2k2
Messages
67
Reaction score
0

Homework Statement


Let f: R-->R be continuous. For δ>0, define g: R-->R by:

g(x) = (1/2δ) ∫ (from x-δ to x+δ) f

Show:

a) g is continuously differentiable

b) If f is uniformly continuous, then, for every ε>0, there exists a δ1>0 such that sup{∣f(x) - g(x)∣; x∈R} < ε for 0<δ≤δ1

The Attempt at a Solution



Please help me, I am confused
 
Last edited:
Physics news on Phys.org
cooljosh2k2 said:

Homework Statement


Let f: R-->R be continuous. For δ>0, define g: R-->R by:

g(x) = (1/2δ) ∫ (from x-δ to x+δ) f
Frankly, I don't understand this part. If f is not linear, or at least piecewise-linear, g will depend upon both x and \delta. Was there a "\lim_{\delta\to 0}" in the definition?

Show:

a) g is continuously differentiable

b) If f is uniformly continuous, then, for every ε>0, there exists a δ1>0 such that sup{∣f(x) - g(x)∣; x∈R} < ε for 0<δ≤δ1


The Attempt at a Solution



For part a, according to the fundamental theorem of calculus, if i integrate the given integral, i get a value of 1. How do i show that g is continuously differentiable though.
?? How do you arrive at the conclusion that the integral is identically 1? The value of the integral will strongly depend upon f.

and for part b, I am just trying to figure out part a before tackling part b.
 
No, there was no limit defined in the problem, i typed it exactly how my prof. worded it.

And you are right about my conclusion for the integral, had a brain cramp and figured that f is equal to 1, which would make my integrating very easy. So completely disregard that i said that.
 
Question: A clock's minute hand has length 4 and its hour hand has length 3. What is the distance between the tips at the moment when it is increasing most rapidly?(Putnam Exam Question) Answer: Making assumption that both the hands moves at constant angular velocities, the answer is ## \sqrt{7} .## But don't you think this assumption is somewhat doubtful and wrong?

Similar threads

Replies
4
Views
4K
Replies
30
Views
3K
  • · Replies 6 ·
Replies
6
Views
2K
Replies
26
Views
2K
  • · Replies 8 ·
Replies
8
Views
2K
Replies
7
Views
1K
  • · Replies 11 ·
Replies
11
Views
2K
  • · Replies 23 ·
Replies
23
Views
2K
  • · Replies 20 ·
Replies
20
Views
3K
Replies
5
Views
2K